Showing a sum is positiveFinding Binomial expansion of a radicalSimplify the Expression $sum _ k=0 ^ n binomnki^k3^k-n $Alternative combinatorial proof for $sumlimits_r=0^nbinomnrbinomm+rn=sumlimits_r=0^nbinomnrbinommr2^r$Proof by induction, binomial coefficientApproximating a binomial sum over a simplexHow to expand $sqrtx^6+1$ using Maclaurin's seriesSum of $m choose j$ multiplied by $2^2^j$How to show that $sumlimits_k=0^n (-1)^ktfrac nchoosek x+kchoosek = fracxx+n$Finite sum with inverse binomialShowing an alternating sum is positive

What does Apple's new App Store requirement mean

Mimic lecturing on blackboard, facing audience

It grows, but water kills it

Does the Linux kernel need a file system to run?

What's the name of the logical fallacy where a debater extends a statement far beyond the original statement to make it true?

Taxes on Dividends in a Roth IRA

How to convince somebody that he is fit for something else, but not this job?

Will number of steps recorded on FitBit/any fitness tracker add up distance in PokemonGo?

How do I fix the group tension caused by my character stealing and possibly killing without provocation?

awk assign to multiple variables at once

Why can't the Brexit deadlock in the UK parliament be solved with a plurality vote?

Stack Interview Code methods made from class Node and Smart Pointers

How would you translate "more" for use as an interface button?

The Digit Triangles

Quoting Keynes in a lecture

Why Shazam when there is already Superman?

Can a stoichiometric mixture of oxygen and methane exist as a liquid at standard pressure and some (low) temperature?

Which Article Helped Get Rid of Technobabble in RPGs?

What is the English pronunciation of "pain au chocolat"?

Does "he squandered his car on drink" sound natural?

Giving feedback to someone without sounding prejudiced

Showing a sum is positive

What is the highest possible scrabble score for placing a single tile

Is it allowed to activate the ability of multiple planeswalkers in a single turn?



Showing a sum is positive


Finding Binomial expansion of a radicalSimplify the Expression $sum _ k=0 ^ n binomnki^k3^k-n $Alternative combinatorial proof for $sumlimits_r=0^nbinomnrbinomm+rn=sumlimits_r=0^nbinomnrbinommr2^r$Proof by induction, binomial coefficientApproximating a binomial sum over a simplexHow to expand $sqrtx^6+1$ using Maclaurin's seriesSum of $m choose j$ multiplied by $2^2^j$How to show that $sumlimits_k=0^n (-1)^ktfrac nchoosek x+kchoosek = fracxx+n$Finite sum with inverse binomialShowing an alternating sum is positive













3












$begingroup$



Show that the sum$$sum_k=0^n n choose kfrac(-1)^kn+k+1$$ is a positive rational number.




It is easy to show that it is a rational number. But I am having trouble showing that this expression is positive. It might be some binomial expansion that I could not get.










share|cite|improve this question











$endgroup$







  • 1




    $begingroup$
    Have you tried using induction on $n$ for example?
    $endgroup$
    – Minus One-Twelfth
    1 hour ago
















3












$begingroup$



Show that the sum$$sum_k=0^n n choose kfrac(-1)^kn+k+1$$ is a positive rational number.




It is easy to show that it is a rational number. But I am having trouble showing that this expression is positive. It might be some binomial expansion that I could not get.










share|cite|improve this question











$endgroup$







  • 1




    $begingroup$
    Have you tried using induction on $n$ for example?
    $endgroup$
    – Minus One-Twelfth
    1 hour ago














3












3








3


1



$begingroup$



Show that the sum$$sum_k=0^n n choose kfrac(-1)^kn+k+1$$ is a positive rational number.




It is easy to show that it is a rational number. But I am having trouble showing that this expression is positive. It might be some binomial expansion that I could not get.










share|cite|improve this question











$endgroup$





Show that the sum$$sum_k=0^n n choose kfrac(-1)^kn+k+1$$ is a positive rational number.




It is easy to show that it is a rational number. But I am having trouble showing that this expression is positive. It might be some binomial expansion that I could not get.







combinatorics summation binomial-coefficients binomial-ideals






share|cite|improve this question















share|cite|improve this question













share|cite|improve this question




share|cite|improve this question








edited 1 hour ago







Hitendra Kumar

















asked 1 hour ago









Hitendra KumarHitendra Kumar

656




656







  • 1




    $begingroup$
    Have you tried using induction on $n$ for example?
    $endgroup$
    – Minus One-Twelfth
    1 hour ago













  • 1




    $begingroup$
    Have you tried using induction on $n$ for example?
    $endgroup$
    – Minus One-Twelfth
    1 hour ago








1




1




$begingroup$
Have you tried using induction on $n$ for example?
$endgroup$
– Minus One-Twelfth
1 hour ago





$begingroup$
Have you tried using induction on $n$ for example?
$endgroup$
– Minus One-Twelfth
1 hour ago











3 Answers
3






active

oldest

votes


















6












$begingroup$

Direct proof:
$$beginsplit
sum_k=0^n nchoose kfrac(-1)^kn+k+1 &=sum_k=0^n nchoose k(-1)^kint_0^1 x^n+kdx\
&=int_0^1x^nsum_k=0^n nchoose k(-x)^kdx\
&=int_0^1x^n(1-x)^ndx
endsplit$$

The latter is clearly a positive number.






share|cite|improve this answer









$endgroup$












  • $begingroup$
    Thanks,I got it.
    $endgroup$
    – Hitendra Kumar
    58 mins ago










  • $begingroup$
    You're welcome!
    $endgroup$
    – Stefan Lafon
    57 mins ago










  • $begingroup$
    How did you conclude that the sum is a limited integral? Do you know where can I find more on this on-line? Thanks.
    $endgroup$
    – NoChance
    56 mins ago






  • 2




    $begingroup$
    It's a "known trick" that $frac 1 p+1 = int_0^1x^pdx$. Then I noticed that the sum looked almost like that of the binomial theorem.
    $endgroup$
    – Stefan Lafon
    51 mins ago










  • $begingroup$
    Thanks for responding.Got it.
    $endgroup$
    – NoChance
    41 mins ago



















3












$begingroup$

When $k=0$ the term is positive. When $k=1$ the term is negative BUT SMALLER (in absolute value) THAN THE $k=0$ TERM.



When $k=2$ the term is positive. When $k=3$ the term is negative BUT SMALLER (in absolute value) THAN THE $k=2$ TERM.



.....



Get it?






share|cite|improve this answer









$endgroup$












  • $begingroup$
    sorry, I did not write question correctly. Now, I have corrected that. By looking at your answer I realized my mistake. Thanks
    $endgroup$
    – Hitendra Kumar
    1 hour ago


















1












$begingroup$

We can specifically prove that
$$
boxedsum_k=0^n n choose kfrac(-1)^kn+k+1=left((2n+1)binom2nnright)^-1
$$

To see this, shuffle a deck of $2n+1$ cards numbered $1$ to $n$. Consider this:




What is the probability that card number $n+1$ is in the middle of the deck, and cards numbered $1$ to $n$ are below it?




The easy answer is the fraction on the RHS. The LHS can be interpreted as an application of the principle of inclusion exclusion. Namely, we first take the probability that card number of $n+1$ is the lowest of the cards numbered $n+1,n+2,dots,2n+1$. This is the $k=0$ term. From this, for each $i=1,dots,n$, we subtract the probability that $n+1$ is the lowest of the list $i,n+1,n+2,dots,2n+1$. This is a bad event, because we want $n+1$ to be above $i$. Doing this for each $i$, we subtract $binomn1frac1n+2$. We then must add back in the doubly subtracted events, subtract the triple intersections, and so on, eventually ending with the alternating sum on the left.






share|cite|improve this answer









$endgroup$












    Your Answer





    StackExchange.ifUsing("editor", function ()
    return StackExchange.using("mathjaxEditing", function ()
    StackExchange.MarkdownEditor.creationCallbacks.add(function (editor, postfix)
    StackExchange.mathjaxEditing.prepareWmdForMathJax(editor, postfix, [["$", "$"], ["\\(","\\)"]]);
    );
    );
    , "mathjax-editing");

    StackExchange.ready(function()
    var channelOptions =
    tags: "".split(" "),
    id: "69"
    ;
    initTagRenderer("".split(" "), "".split(" "), channelOptions);

    StackExchange.using("externalEditor", function()
    // Have to fire editor after snippets, if snippets enabled
    if (StackExchange.settings.snippets.snippetsEnabled)
    StackExchange.using("snippets", function()
    createEditor();
    );

    else
    createEditor();

    );

    function createEditor()
    StackExchange.prepareEditor(
    heartbeatType: 'answer',
    autoActivateHeartbeat: false,
    convertImagesToLinks: true,
    noModals: true,
    showLowRepImageUploadWarning: true,
    reputationToPostImages: 10,
    bindNavPrevention: true,
    postfix: "",
    imageUploader:
    brandingHtml: "Powered by u003ca class="icon-imgur-white" href="https://imgur.com/"u003eu003c/au003e",
    contentPolicyHtml: "User contributions licensed under u003ca href="https://creativecommons.org/licenses/by-sa/3.0/"u003ecc by-sa 3.0 with attribution requiredu003c/au003e u003ca href="https://stackoverflow.com/legal/content-policy"u003e(content policy)u003c/au003e",
    allowUrls: true
    ,
    noCode: true, onDemand: true,
    discardSelector: ".discard-answer"
    ,immediatelyShowMarkdownHelp:true
    );



    );













    draft saved

    draft discarded


















    StackExchange.ready(
    function ()
    StackExchange.openid.initPostLogin('.new-post-login', 'https%3a%2f%2fmath.stackexchange.com%2fquestions%2f3157740%2fshowing-a-sum-is-positive%23new-answer', 'question_page');

    );

    Post as a guest















    Required, but never shown

























    3 Answers
    3






    active

    oldest

    votes








    3 Answers
    3






    active

    oldest

    votes









    active

    oldest

    votes






    active

    oldest

    votes









    6












    $begingroup$

    Direct proof:
    $$beginsplit
    sum_k=0^n nchoose kfrac(-1)^kn+k+1 &=sum_k=0^n nchoose k(-1)^kint_0^1 x^n+kdx\
    &=int_0^1x^nsum_k=0^n nchoose k(-x)^kdx\
    &=int_0^1x^n(1-x)^ndx
    endsplit$$

    The latter is clearly a positive number.






    share|cite|improve this answer









    $endgroup$












    • $begingroup$
      Thanks,I got it.
      $endgroup$
      – Hitendra Kumar
      58 mins ago










    • $begingroup$
      You're welcome!
      $endgroup$
      – Stefan Lafon
      57 mins ago










    • $begingroup$
      How did you conclude that the sum is a limited integral? Do you know where can I find more on this on-line? Thanks.
      $endgroup$
      – NoChance
      56 mins ago






    • 2




      $begingroup$
      It's a "known trick" that $frac 1 p+1 = int_0^1x^pdx$. Then I noticed that the sum looked almost like that of the binomial theorem.
      $endgroup$
      – Stefan Lafon
      51 mins ago










    • $begingroup$
      Thanks for responding.Got it.
      $endgroup$
      – NoChance
      41 mins ago
















    6












    $begingroup$

    Direct proof:
    $$beginsplit
    sum_k=0^n nchoose kfrac(-1)^kn+k+1 &=sum_k=0^n nchoose k(-1)^kint_0^1 x^n+kdx\
    &=int_0^1x^nsum_k=0^n nchoose k(-x)^kdx\
    &=int_0^1x^n(1-x)^ndx
    endsplit$$

    The latter is clearly a positive number.






    share|cite|improve this answer









    $endgroup$












    • $begingroup$
      Thanks,I got it.
      $endgroup$
      – Hitendra Kumar
      58 mins ago










    • $begingroup$
      You're welcome!
      $endgroup$
      – Stefan Lafon
      57 mins ago










    • $begingroup$
      How did you conclude that the sum is a limited integral? Do you know where can I find more on this on-line? Thanks.
      $endgroup$
      – NoChance
      56 mins ago






    • 2




      $begingroup$
      It's a "known trick" that $frac 1 p+1 = int_0^1x^pdx$. Then I noticed that the sum looked almost like that of the binomial theorem.
      $endgroup$
      – Stefan Lafon
      51 mins ago










    • $begingroup$
      Thanks for responding.Got it.
      $endgroup$
      – NoChance
      41 mins ago














    6












    6








    6





    $begingroup$

    Direct proof:
    $$beginsplit
    sum_k=0^n nchoose kfrac(-1)^kn+k+1 &=sum_k=0^n nchoose k(-1)^kint_0^1 x^n+kdx\
    &=int_0^1x^nsum_k=0^n nchoose k(-x)^kdx\
    &=int_0^1x^n(1-x)^ndx
    endsplit$$

    The latter is clearly a positive number.






    share|cite|improve this answer









    $endgroup$



    Direct proof:
    $$beginsplit
    sum_k=0^n nchoose kfrac(-1)^kn+k+1 &=sum_k=0^n nchoose k(-1)^kint_0^1 x^n+kdx\
    &=int_0^1x^nsum_k=0^n nchoose k(-x)^kdx\
    &=int_0^1x^n(1-x)^ndx
    endsplit$$

    The latter is clearly a positive number.







    share|cite|improve this answer












    share|cite|improve this answer



    share|cite|improve this answer










    answered 1 hour ago









    Stefan LafonStefan Lafon

    3,02019




    3,02019











    • $begingroup$
      Thanks,I got it.
      $endgroup$
      – Hitendra Kumar
      58 mins ago










    • $begingroup$
      You're welcome!
      $endgroup$
      – Stefan Lafon
      57 mins ago










    • $begingroup$
      How did you conclude that the sum is a limited integral? Do you know where can I find more on this on-line? Thanks.
      $endgroup$
      – NoChance
      56 mins ago






    • 2




      $begingroup$
      It's a "known trick" that $frac 1 p+1 = int_0^1x^pdx$. Then I noticed that the sum looked almost like that of the binomial theorem.
      $endgroup$
      – Stefan Lafon
      51 mins ago










    • $begingroup$
      Thanks for responding.Got it.
      $endgroup$
      – NoChance
      41 mins ago

















    • $begingroup$
      Thanks,I got it.
      $endgroup$
      – Hitendra Kumar
      58 mins ago










    • $begingroup$
      You're welcome!
      $endgroup$
      – Stefan Lafon
      57 mins ago










    • $begingroup$
      How did you conclude that the sum is a limited integral? Do you know where can I find more on this on-line? Thanks.
      $endgroup$
      – NoChance
      56 mins ago






    • 2




      $begingroup$
      It's a "known trick" that $frac 1 p+1 = int_0^1x^pdx$. Then I noticed that the sum looked almost like that of the binomial theorem.
      $endgroup$
      – Stefan Lafon
      51 mins ago










    • $begingroup$
      Thanks for responding.Got it.
      $endgroup$
      – NoChance
      41 mins ago
















    $begingroup$
    Thanks,I got it.
    $endgroup$
    – Hitendra Kumar
    58 mins ago




    $begingroup$
    Thanks,I got it.
    $endgroup$
    – Hitendra Kumar
    58 mins ago












    $begingroup$
    You're welcome!
    $endgroup$
    – Stefan Lafon
    57 mins ago




    $begingroup$
    You're welcome!
    $endgroup$
    – Stefan Lafon
    57 mins ago












    $begingroup$
    How did you conclude that the sum is a limited integral? Do you know where can I find more on this on-line? Thanks.
    $endgroup$
    – NoChance
    56 mins ago




    $begingroup$
    How did you conclude that the sum is a limited integral? Do you know where can I find more on this on-line? Thanks.
    $endgroup$
    – NoChance
    56 mins ago




    2




    2




    $begingroup$
    It's a "known trick" that $frac 1 p+1 = int_0^1x^pdx$. Then I noticed that the sum looked almost like that of the binomial theorem.
    $endgroup$
    – Stefan Lafon
    51 mins ago




    $begingroup$
    It's a "known trick" that $frac 1 p+1 = int_0^1x^pdx$. Then I noticed that the sum looked almost like that of the binomial theorem.
    $endgroup$
    – Stefan Lafon
    51 mins ago












    $begingroup$
    Thanks for responding.Got it.
    $endgroup$
    – NoChance
    41 mins ago





    $begingroup$
    Thanks for responding.Got it.
    $endgroup$
    – NoChance
    41 mins ago












    3












    $begingroup$

    When $k=0$ the term is positive. When $k=1$ the term is negative BUT SMALLER (in absolute value) THAN THE $k=0$ TERM.



    When $k=2$ the term is positive. When $k=3$ the term is negative BUT SMALLER (in absolute value) THAN THE $k=2$ TERM.



    .....



    Get it?






    share|cite|improve this answer









    $endgroup$












    • $begingroup$
      sorry, I did not write question correctly. Now, I have corrected that. By looking at your answer I realized my mistake. Thanks
      $endgroup$
      – Hitendra Kumar
      1 hour ago















    3












    $begingroup$

    When $k=0$ the term is positive. When $k=1$ the term is negative BUT SMALLER (in absolute value) THAN THE $k=0$ TERM.



    When $k=2$ the term is positive. When $k=3$ the term is negative BUT SMALLER (in absolute value) THAN THE $k=2$ TERM.



    .....



    Get it?






    share|cite|improve this answer









    $endgroup$












    • $begingroup$
      sorry, I did not write question correctly. Now, I have corrected that. By looking at your answer I realized my mistake. Thanks
      $endgroup$
      – Hitendra Kumar
      1 hour ago













    3












    3








    3





    $begingroup$

    When $k=0$ the term is positive. When $k=1$ the term is negative BUT SMALLER (in absolute value) THAN THE $k=0$ TERM.



    When $k=2$ the term is positive. When $k=3$ the term is negative BUT SMALLER (in absolute value) THAN THE $k=2$ TERM.



    .....



    Get it?






    share|cite|improve this answer









    $endgroup$



    When $k=0$ the term is positive. When $k=1$ the term is negative BUT SMALLER (in absolute value) THAN THE $k=0$ TERM.



    When $k=2$ the term is positive. When $k=3$ the term is negative BUT SMALLER (in absolute value) THAN THE $k=2$ TERM.



    .....



    Get it?







    share|cite|improve this answer












    share|cite|improve this answer



    share|cite|improve this answer










    answered 1 hour ago









    David G. StorkDavid G. Stork

    11.1k41432




    11.1k41432











    • $begingroup$
      sorry, I did not write question correctly. Now, I have corrected that. By looking at your answer I realized my mistake. Thanks
      $endgroup$
      – Hitendra Kumar
      1 hour ago
















    • $begingroup$
      sorry, I did not write question correctly. Now, I have corrected that. By looking at your answer I realized my mistake. Thanks
      $endgroup$
      – Hitendra Kumar
      1 hour ago















    $begingroup$
    sorry, I did not write question correctly. Now, I have corrected that. By looking at your answer I realized my mistake. Thanks
    $endgroup$
    – Hitendra Kumar
    1 hour ago




    $begingroup$
    sorry, I did not write question correctly. Now, I have corrected that. By looking at your answer I realized my mistake. Thanks
    $endgroup$
    – Hitendra Kumar
    1 hour ago











    1












    $begingroup$

    We can specifically prove that
    $$
    boxedsum_k=0^n n choose kfrac(-1)^kn+k+1=left((2n+1)binom2nnright)^-1
    $$

    To see this, shuffle a deck of $2n+1$ cards numbered $1$ to $n$. Consider this:




    What is the probability that card number $n+1$ is in the middle of the deck, and cards numbered $1$ to $n$ are below it?




    The easy answer is the fraction on the RHS. The LHS can be interpreted as an application of the principle of inclusion exclusion. Namely, we first take the probability that card number of $n+1$ is the lowest of the cards numbered $n+1,n+2,dots,2n+1$. This is the $k=0$ term. From this, for each $i=1,dots,n$, we subtract the probability that $n+1$ is the lowest of the list $i,n+1,n+2,dots,2n+1$. This is a bad event, because we want $n+1$ to be above $i$. Doing this for each $i$, we subtract $binomn1frac1n+2$. We then must add back in the doubly subtracted events, subtract the triple intersections, and so on, eventually ending with the alternating sum on the left.






    share|cite|improve this answer









    $endgroup$

















      1












      $begingroup$

      We can specifically prove that
      $$
      boxedsum_k=0^n n choose kfrac(-1)^kn+k+1=left((2n+1)binom2nnright)^-1
      $$

      To see this, shuffle a deck of $2n+1$ cards numbered $1$ to $n$. Consider this:




      What is the probability that card number $n+1$ is in the middle of the deck, and cards numbered $1$ to $n$ are below it?




      The easy answer is the fraction on the RHS. The LHS can be interpreted as an application of the principle of inclusion exclusion. Namely, we first take the probability that card number of $n+1$ is the lowest of the cards numbered $n+1,n+2,dots,2n+1$. This is the $k=0$ term. From this, for each $i=1,dots,n$, we subtract the probability that $n+1$ is the lowest of the list $i,n+1,n+2,dots,2n+1$. This is a bad event, because we want $n+1$ to be above $i$. Doing this for each $i$, we subtract $binomn1frac1n+2$. We then must add back in the doubly subtracted events, subtract the triple intersections, and so on, eventually ending with the alternating sum on the left.






      share|cite|improve this answer









      $endgroup$















        1












        1








        1





        $begingroup$

        We can specifically prove that
        $$
        boxedsum_k=0^n n choose kfrac(-1)^kn+k+1=left((2n+1)binom2nnright)^-1
        $$

        To see this, shuffle a deck of $2n+1$ cards numbered $1$ to $n$. Consider this:




        What is the probability that card number $n+1$ is in the middle of the deck, and cards numbered $1$ to $n$ are below it?




        The easy answer is the fraction on the RHS. The LHS can be interpreted as an application of the principle of inclusion exclusion. Namely, we first take the probability that card number of $n+1$ is the lowest of the cards numbered $n+1,n+2,dots,2n+1$. This is the $k=0$ term. From this, for each $i=1,dots,n$, we subtract the probability that $n+1$ is the lowest of the list $i,n+1,n+2,dots,2n+1$. This is a bad event, because we want $n+1$ to be above $i$. Doing this for each $i$, we subtract $binomn1frac1n+2$. We then must add back in the doubly subtracted events, subtract the triple intersections, and so on, eventually ending with the alternating sum on the left.






        share|cite|improve this answer









        $endgroup$



        We can specifically prove that
        $$
        boxedsum_k=0^n n choose kfrac(-1)^kn+k+1=left((2n+1)binom2nnright)^-1
        $$

        To see this, shuffle a deck of $2n+1$ cards numbered $1$ to $n$. Consider this:




        What is the probability that card number $n+1$ is in the middle of the deck, and cards numbered $1$ to $n$ are below it?




        The easy answer is the fraction on the RHS. The LHS can be interpreted as an application of the principle of inclusion exclusion. Namely, we first take the probability that card number of $n+1$ is the lowest of the cards numbered $n+1,n+2,dots,2n+1$. This is the $k=0$ term. From this, for each $i=1,dots,n$, we subtract the probability that $n+1$ is the lowest of the list $i,n+1,n+2,dots,2n+1$. This is a bad event, because we want $n+1$ to be above $i$. Doing this for each $i$, we subtract $binomn1frac1n+2$. We then must add back in the doubly subtracted events, subtract the triple intersections, and so on, eventually ending with the alternating sum on the left.







        share|cite|improve this answer












        share|cite|improve this answer



        share|cite|improve this answer










        answered 22 mins ago









        Mike EarnestMike Earnest

        25.5k22151




        25.5k22151



























            draft saved

            draft discarded
















































            Thanks for contributing an answer to Mathematics Stack Exchange!


            • Please be sure to answer the question. Provide details and share your research!

            But avoid


            • Asking for help, clarification, or responding to other answers.

            • Making statements based on opinion; back them up with references or personal experience.

            Use MathJax to format equations. MathJax reference.


            To learn more, see our tips on writing great answers.




            draft saved


            draft discarded














            StackExchange.ready(
            function ()
            StackExchange.openid.initPostLogin('.new-post-login', 'https%3a%2f%2fmath.stackexchange.com%2fquestions%2f3157740%2fshowing-a-sum-is-positive%23new-answer', 'question_page');

            );

            Post as a guest















            Required, but never shown





















































            Required, but never shown














            Required, but never shown












            Required, but never shown







            Required, but never shown

































            Required, but never shown














            Required, but never shown












            Required, but never shown







            Required, but never shown







            Popular posts from this blog

            Magento 2 duplicate PHPSESSID cookie when using session_start() in custom php scriptMagento 2: User cant logged in into to account page, no error showing!Magento duplicate on subdomainGrabbing storeview from cookie (after using language selector)How do I run php custom script on magento2Magento 2: Include PHP script in headerSession lock after using Cm_RedisSessionscript php to update stockMagento set cookie popupMagento 2 session id cookie - where to find it?How to import Configurable product from csv with custom attributes using php scriptMagento 2 run custom PHP script

            Can not update quote_id field of “quote_item” table magento 2Magento 2.1 - We can't remove the item. (Shopping Cart doesnt allow us to remove items before becomes empty)Add value for custom quote item attribute using REST apiREST API endpoint v1/carts/cartId/items always returns error messageCorrect way to save entries to databaseHow to remove all associated quote objects of a customer completelyMagento 2 - Save value from custom input field to quote_itemGet quote_item data using quote id and product id filter in Magento 2How to set additional data to quote_item table from controller in Magento 2?What is the purpose of additional_data column in quote_item table in magento2Set Custom Price to Quote item magento2 from controller

            How to solve knockout JS error in Magento 2 Planned maintenance scheduled April 23, 2019 at 23:30 UTC (7:30pm US/Eastern) Announcing the arrival of Valued Associate #679: Cesar Manara Unicorn Meta Zoo #1: Why another podcast?(Magento2) knockout.js:3012 Uncaught ReferenceError: Unable to process bindingUnable to process binding Knockout.js magento 2Cannot read property `scopeLabel` of undefined on Product Detail PageCan't get Customer Data on frontend in Magento 2Magento2 Order Summary - unable to process bindingKO templates are not loading in Magento 2.1 applicationgetting knockout js error magento 2Product grid not load -— Unable to process binding Knockout.js magento 2Product form not loaded in magento2Uncaught ReferenceError: Unable to process binding “if: function()return (isShowLegend()) ” magento 2